You are on page 1of 102
FOREWORDS: Most of the exercises contained herein are taken from W.Rudin's treatise "Real and Complex Analysis." (Mac Graw Hill, N.Y. 1966 § 1974). Exercises Peeler Z have drawn from several sources; among these, Hewitt & in Stromberg's treatise “Real and Abstract Analysis " : (Springer Verlag N.Y. 1965) must be singled out. Real Analysis we gin with vant, Solutions BY A GROUP OF GRADUATE STUDENTS UNIVERSITY OF SAIGON | como sr Vti Trong Tuan Throughout his chapcer, p Ls a positive measure ono mecsurabte space (XML) and bg) be sequences ix [o0, 0]. Prove exencase 1.- tet | the fottovinge (a) Lom sup (o2,) = = Lim inf (b) Lin aup (a,¢b,) © Lim sup a, + Lim oup by, ol If a, <2, for ait n, then Lim inf a, & Lim tn 5, Shos by an example that atrtab inaquatity ean hotd in (2). souirtow Go). (0), (6) are rly, S48 for exapte (@- Sisk poke aatinag enleixeale cint.ns 2c Ry 37 ‘repeated applications of this identity. we have Lim sup (-a,) * inf sup (a,) + inf Ciné 2) Tape mabe sesup ing a, + “Liman a. ko apk kok We can take the following example : let a,e(-1)",d,*(-"? ‘Then Lin sup a, = Lin sup B, = 1, but Lin ( yt By) OSLO TS Lin sup a, + Lin sup by ‘Therefore, strict inequality can hold in (b). 1 fe even, DAERCISE 2.- t fp e Ey Pm ee Olt, eT He SOLUTION t isa unset fiegan cain fennel. ris shove Ula in Fatou's Lemna ve may have strtee snequaticy. yr that Lim inf f,* 0, and that 4 , ten Sean ‘ GXERCISE 3.- suppoue J, : X—HL0,00] te fy > fy> Sqlt)—fle) a2 ne, ner, Suppose f, € 170 p), prove that then aeurabte, + 0 and SE tn. show thas ence eonotuaton doas not hold if *f; ¢ I7( KI" te onttted, souuTtoM bat f be in Lt, ana tet £, 09 }etn), xan the conenue sion follows fron Lebesgue's dosinated convergence theoren, since lls najortzed by £,. Now suppose the ci ‘he foltoving exampte: fd Foot es uniformly to 9, but dition " £,@ LI" is onitted. Consider Then £, decres Ey (e)oe > ya 1,2,, EXERCISE 4.- Prove chat the eet of points at uhioh « sequence of macouradi« real funstions sonvergee te a measurable oo? . sowrtox bet (f,) be a sequence of real measurable functions, Then,. 4m Lin sup £, and k= tim inf £, are acasurabie, Recall that a seqionce’of real muabers (4,) converges i¢ and only 4f lin sup a, = Lim inf a, , hence Lin (5) exists in {-0yo]) 466 g(x} + Ge). Since g and h are Reasurabio, the Ix + £0) converges{ = fx: etx) = Rox} ds a This completes the proof, EXERCISE = tat £ be an enccuntabte set, end te2 = {4 ex: atther 4 07 A ta at sont sountadte YP “eetine “was S047 4 te oe nove souneasie, MA) © 1 tf 4° te at most sountable G00 thee CAM in « onatgetre, endn ta e measure on GR so.urtow det Getacx: A ts at most countable i G-1N CX : Ae As at most countabte|. Since X is uncountable, Gand G are disjoint, hence {Gg | form a partition of Cm. By definition of F, ve have $e; Ac ant se smpiy that the sets A UB, AB and A - B all Belong to, Since Geonsists exactly of all A with ASe G, ve have X€@ j Ae G and 3G inply that the sets AN3, A. U3 and all belong tog. Now suppose Ae Gand 86. Then the follovings hold + ANBEA, hence AN EG umes An ae cae, thus AUD EG, hence UBeg Therefcre we have a ee; ream; (seGitand sean) (4 U8, An, and A-8 betong co OR! 8 Now let A, €QMn = 1,2, vomaas Ua. If A, As at most countable for all a, then Ais at most countable, hace Ae GF. TEA, © G for sone ne, then at = Maz cas, with ant oA, ASEH s dence Me GF, hence ae @. We conclude that eG metytyicses, inption Ta, setonge race (1) and @) show that Mis a ™B agebra on t, By definition of w, (A) = 01€ Ae G and s *Q) oi neg, In particular (a) = 0 and Gx) = 1. Let us prove that pis completely additive, First note that AE AGG and De Grthen AUB is a proper subset of X, Hence if A © G and B is disjoint from A, then Be GF. Let (A,) be a partition of A, with Aje QM; n-t,2, TE AGG, then A, © G for all a, Hence oy Sa since HOA) = 85m = 1,2y000 If A € Gy then there is sone ne with A, € G (For other wise 6G). By the preceding remark, for n # At ve have Aye GF wd ste wAr= was Dwar: Leap. Fhe i Thus 4s completely additive, hence p is s measure on the measurable space (,QW). QED. EXERCISE bow tat Wlbe a a~ atgedra on a set I. Suppose lie ab ‘moet countabte, prove’ that@M. has 2 elemente for some integer ne SOLUTION Suppose CWLis at most countable, Define an equivalence relation es follows : (amy) tte (xe inti ye fr 4 ct) Then the equivalence class ¥ Sf x is the intersection of alt measurable sets containing x. Since there are st nost countably many measurable sets in x, ths inplies that ¥ is measurable, Let F be the quotient space X/y = FEOM, hence F is at most countable. Define eM —- De) by vu) = {Fr xeM], Then the foltovings hold : F fs at'most countable, hence ¢ is surjective, : XEN, M, teplies,x CM, = M,, for My and M, in, so ate is infective tn! eT ant ty OR Thus 4 1s Difective, If Gillis finite, then F is finite and has n elenente, watch implies that QC) nas 2” elenents, and hence chat Qivhas 7 clenents, ’ Suppose QW is countabte, By the foregoing result, F te countable, hence F is equipotent tN; but QIN) is uncountable (See Remark below), and 4:Q —~ QU) ts a bijection, # contre: dlction, Thus F ix finite, and the preceding case occurs.d.6.D. RETUNK.= Por avery set, 10 act equtpotent to Dit). Indeed p12 £4 © DU) tee napping, Let 5 as[rerr rec}. Then A ECE). Tate shove that Card(e) < easel DUE)). 20 the case Ew Wy, note tna Din) 2 agvizotent to the set Nora[™ of att mappings +s —-o/t|, and the latter 19 eavipocent fo the tetaate cantor sets rinally, ve have card{DIn)) = careiB)« See zxeretse [Te] (a) end (0), ch. Witte EXERCISE 7. Suppose —plt)cea , (fq) 2 a 2 complex measurable functions on I, and f, — f uniformly on x. | Prove that tan frame fran. ed )! and thov that the hypotheeia " plT/ ne, HE,GOI1 + If, (DI for 4x and n> mee Since £, 4s bounded and p(X) finite, 1+ If, | ds integrable on x, By Lebergue's doninated convergence ‘theorem , the result now follows . Next, tot X= (0,eof, and £, = 2% q,q)+ Then (4) con verges uniformly to 0 on X, but [Fae reraeta I This shows that the hypothesis " w(X)<- " cannot be onitted. QED. EXERCISE Bon cet By t= Tas Eafe ex.s betonge 20 insintzety many ¥,} fe at Prove that B= Ae = im at moat finttaly many of the sete By. souuTrow ‘ mine Daim aoa ceca Ae Let x be in E, Then for all a, x @ A for sone k >A. Thus : xe A, for alla, hence xe A. 10 Conversely, if x € A then x @A, for all n, hence xeE, k(n) for some K(n) > n. This implies x belongs to infinitely many 5,. Ths B= A QED, extere = PRERCISE tex cae ¢ 14) Hy, Prove that to each E> 6 the: ae eth [lslen 0, there exists an as such that ¥< fast-np0 < fan J . Since s,s simple and integrable, there is an W <= such’ that + de Een, 506 ne, he Then 6 HO) <8, ve have fier < fon = ide + foam < fe + s,)ée (me ode le < frgee Pat a EXERCISE 10.~ suppose p te = positive mecoure on Ky f+ X —~ Coe} ta meceurabie , f° fax = 2, where 0 <0 < wm, and ais a constont [ni eeees ie fone (SJ = ete ae beg ices sours Wat WO) + te @ +t Ste Me Ht) = 8, ED = oe sad (0) = tim Fath) = 1, Tis gives we limaAler a+ Psu w>0) Take A= n, this gives @) linn og G2 Dew Take A= n@,a>0, then A—eoo as neo, Thus Rim nt ert, ee asa ees foc e <1, ates (1+) - ue 1 0 ‘Thus ve have @) tin m tog (2+ (2)") = 0 lee ce G) tise tes (1+ ()') += @ 0] has nonzero measure, since [tap > 0. fe: fe ® @), OM linge xek (OJote = Ge) > 0). Tt follows that iim inf [gna ==, hence tim fonds = Tus @) tin, fa tor (1 + (f)") abe = @ <« ©, and fae of centers a, C14) Aig, Mn mesure $n, nd S54 ton at 2 open aa Aa MEP Len Ea one can see the followings (2) Gi) holds veh n= 2+ (b) ©1, hence Af statement (444) holds, then statement GGA) holds with (n + 1) instead of a. (©) By (ii), the A,'s are mutualay disjoint. let B= AU AU. By (48) and (6), a8) Dy R(A,), hetice by (iii), m(E) = € - By (41), any interval vaieh does not intersect B has length SC Epcos +, hence every open set contains point off. oo ‘REMARKS (a) tat £ be a sepmene of 1e0900 4 tiation of as coettiotent , the above construction gives an open set & : fn Ty dense in ty a () Every open subset of # in (a) has 4 nas measure EL eastick, le) 1 =F ia partact ( having 20 szolated point) and contains no nonempty open segment, hence 1 ~ F has ao connected component consisting of moze than one point. (a) a(ter) = ae), EXERCISE 2.- civen 0 GE'<1 , oonotruct « totally deconnested eet Kim [0,1] aueh that (ie) = H1.(K te te have no connected cubset consteting of more than one point), Suppose 0< If 9 te Lover semteontinouel(t.e.8) and oS Ayehou that v0 inplies “%(x)>0, hence %(2) = 1 uo that eK, Since ¥ ds Ls.e, } But K is totally disconnected, hence K contains no open segment Jo, p[ with « 0} a py, hence ¥(0) <0 for #11 x. susie xn «evo f WC0> of ts an open subset of K a> "aa : a leeceupiew asa h ; wv)én > Therefore, z,cannot be approxi functions « QED. “ EXERCISE 3.~ Lat 0 et eh ae eeeeetie +08, AS an open set constructed indueti-- (2) As the open dense subset of (0,1) of measure” &,, constructed as in the solutton to [i]. (i) Each'E, $s the union oF countably many disjoint open intervals I, of length t, k= 1,2y..c.juith vk G4) EL, SB ds such that £4, 1, is open and dense in I, yy has measure constructed as in Renark (4) t let B= BME, n NBA Fr GED, By has measure PRED a, Gyn). Hence (it) is valid with aol instead of a. By (444) and Remark (6) in length < 7) every interyal in 1, has - Moreover, aq, ae 7 is>0, wo Bp, sae [ be an interval in [0,1], Let ng be suck chat 27%<¢ 3 te is ctear that By is dense 9 0. Now let = Je wv so that there exists # point DEE, with IRealcz"e: be, fot some k= 1.20004 but | sotht Tet by (i), act, a, 0) sapites acl, ty 0), atrD ED > 0% By), mA EF) D0, hence ag ab) Sac. Me have therefore proved that cm 0) rf f is Lebesgue measurable,so A is Lebesgue measurabie,vr €Q. Note that ALC AL, If re rt Let fry tps sees [be an enumeration of Q. We choose , by induction on n , a Borel measursbie set A, such that * als, a 3) +0 and such that ea, Te is clear that this choice is possible. if tert, rand rte. rue a) © supfreQix ea} force Ua, HC) = 0, othervise then g 1s Borel neasurabie. By construction, £{x) * g(x) if xe Wa 4B) oN, and W As a set of zero measure, tince fob te uit & ANOTHER SOLUTION For real f, £ = £* = f°, f* and £° are Lebesgue measurable tn, te seticte to ecaides taccare f>> 00 fore isi sagaice (g) of betgeisiegie fesiech cue pee pieae es edsten aif fo-s seats etne eet wee tao MeO EGN igior & Ge He ciel Gn gs eee te Oa, tm 00 = 5,00, carat : 100+ £0 ne QED, EXERCISE [22) sueh thet 0S 7,51 end = Construct « eaquenae of continuous functions {f,) on but suck that the sequence Ss fnerd converges for no #6 10,2) « soLuriow For k= 1,2 sasdedioe 6 bY ay) ator tel< b +0 for Itl> sbutltorgchic ee | a oe nthe graph of oy Now define gy g(t) = g(t = Fp) for tE[0)1, a= 1,2,00,2k. Consider the sequence | fl as It is clear, by construction , that © fain < fexcsyee 0} = ayy Ayu oo with CB) Suppose £ Ss cones : Agee teeoled} mets aie since [¢(s)1 >E for xe A, ye h vig 0 It is clear thet @(V,) <2r, and x.€ V,, so that (4) = 0, Conversely, suppose ot.) * O,ence by definition of 9, for r> 0 there exists @ neighborhood V of x. with ¥(V) 0, addy © Ait eg Pe i te ts clear that fxs of) = 0] As. the sntersection of the A/a = L2yesse, hence dy (B)y the set of points of EXERCISE 8.- Let (8,@) a topological space, (F,d ) a metria EEE aes space, and f1t—+P a mapping wtf VCE, define continuity of £ 4s * ct BAe aca. ry = eapfetrean pesres EY oe Por ee, tet ofa) = tnt | WV): V tu a neighborhood of © : t f 7 EXERCISE “e- bet [J] be a sequence of reat nonnegative functions (a) Prove that ¢ : (2,GI—[0,-0] te upper semi-continuone Re eee oe on R! , and consider the fottowing 16 pe pee (a) If fy ond fy are menses then fytty te toe te continous at potnt = iff ate) «0, (0) Af fy ond fy ore Lomas then tf to (e) show that the ast of pointe of contimitty of f is a G « ape 7 fa nnes sougriow &,, {a} Suppose (2) 0, vith Cc) # £0.) +28 Velen Now for K, and K, in HE, we have PLOK, 0,9] = mE UKE) = 0 hence w(K, K,) = 1,20 thet KL a Kye Re Thus this family Qof closed sets in the coapact space X has the "finite intersection property ",s0 that the intersection K of all nenbers of K, (] rq sis nonempty ,and compact. If V ds an open set containing K, then the collection consisting of V and KS, a @A, fora an open covering of X. X is compact, therefore we can extract a finite subcovering : ux =x vuru 1 This implies KU so that (VW) = Lysince A, ceseeeete AK, belonsste R. ‘This we have proved that q(V) = 1 whene + ¥ ka an open set containing K But p is repular, s0 that ve have wR) = inf | pt): V open ok = 2 The preceding proves the folleving essertions H fs compact in X and H 6 implies KeH H coapact and His 4 proper’ subset of K inplies Hd K. In the second case, He GK, hence pl) # 1yhenee pli) <1 QED a FRERCISE 12.- (a) Shov thet avery compact eubset of RU te the support of @ Soret meareure, (B) If K ta a oloeed eet in a metric space t, show that Kis the aupport of @ continuous function iff X te the cloeure of an open eet, (2) Prove the fottoving genezat ization of (b): If Xia anormal topological apace, then a cloaed eet Xin X te the support of a continuous function f:%—+R iff there astete @ eet 7 and GE) Ven open, (Gt) tua Fe nat, (EEL) tn the ctowure of ¥. sousrtoM (0 For every B= 12,,00 may finds, at, ik with ke . * © Uae, . 3 ne Hobs Therefore we find a sequence feytgeee| = A dn K wih ae We define 4 by the formois p{jx,l) = Zysand for E doret 1, that x, © E.It ts clesr that pfs 4 Borel measure on R° .Yoreorer, for V open in R}, we have (V) > 0 Iff VMK ds nonempty,for A is dense in K, This shows that K is the support of » in the following sense: Kis the smallest closed set in R with the property that w(K°)=0, () Let F be a closed set in the metric space (K, 9). pu) = int | Guy cer Tt is known (exercise) that pp 12 uniformly continuous, pox) = 0 iff xe, Now let K be a closed set in X, If K is the support of ‘the continuous function £:X—+R, then by definition , K is the closure of the set Define ps X—+Rby and that va fx: e040] Waich Ls open jsince £ is continuous. Conversely, suppose K = for V open, Let F =v", and, We see that ppix— Ris continuous , and p(x) + 0 iff x @ F, Rence p(x) > 0 iff xe ¥ % ‘Therefore K+ Vis the closure of the set fx:poo #0} nat is, Kis the support of pp + (e) suppose X is a normal space, and let K be a closed set ink First , if K is the closure of V+ fx: f(x) #0) where €; Xe Ris continuous, then vaW fx: teoole db} - URS each fy 15 closed since £ fs continuous. the sufficlent condition. Conversely suppose ‘Therefore we have proved k is the closure of an open set V, ce sey with F, closedim = 1y2yi.26 verwrue by Urysohn's Lemasa , there exists for every 2 #112 Ee iatu2) with then? thee sper put + Ste, then'ie ds clear thet £ is the unifors continuous function init of the soltince (oy) + with fy 18,7 000004 Bay to that fx —+ 01] isa Coneimueus function. hence If xe V., then there exists ann such that x@F,y f(x) >, EDO. TE xe V then £0.) 20, 0 Byes, Renee £6) + 0 ‘Therefore 2 ty #0}* Ye hence K+ Vis the support of f. nae EXERCISE 22-~ Lee X be a metric apace, vith metric p- any nonempty FC, define aici [weunvette soo thetl py ia a antponaly continuous fanetion on nt that pyle) = 0 Sif = belongs to the olonere Ff apn and A eve dinjotnt fonanpty tioned wubeeoe of Zr sconine the relevance of the funatton : fora $8) = gare ola to Unysohnts Lena, 6 souurtoK Let x,xie x and y @E. ie have PO) < pCR") = plxtyy) (triangle inequatity) = hence pel) < plx.x") = px? ay) Take the infinun of the right side of this inequality as y ranges over E, we obtain fet)“ aa) © mete) vor | mod «mete plea? Iiterchangag the roles of x and x, ve have eG) = Fels) < oben"). ths Tre") = peel < eax"). Therefore, pp is uniformly continuous . Moreover, it is clear thay the following statements are equivalent (4) x betongs’ to the closure F of & Gi) For every e142, such that agen) 0 for all x in x. Therefore is a continuous function of x» per a £0 ts continuous on X [ne = ee Fre crates eee XE AGE py) 20 (iff xe A: 0 that £@) +1 XE BEE pix) = 0 (1 XE Bie, HO) +0 Therefore £ is the function whose existence is asserted in the Urysohn Lema, QED. ar EXERCISE 23.+ Eeantne the proof of Ris 214), and prove the fottoving: (a) If 8, 6 Vy 5 Bye Vy where V, and Vy are dtasoint open sate, then 2! theorem (Sheorem ES, U Ey) = BEE, + B08, Jen Ef By and Rare not in OM, (b) tf 86 CH, then B= HUE, U ky U voce shore J] to @ countadte cottection of joint gonpast sete ad aso, souuTi0N (a) Suppose f, and V, as asserted , 51,2. Te is clear that aC, UE) < mB) + mE) Conversely , 1st V be an open set containing B=, UE, Note that V, and V, are disjoint . Therefore we have FO) DMV AY) + HO MY) & REEL © ACE) Since ve ha HE) + Ink {ACD V opens VO this gives the reversed inequality ME,UE,) > eG) + H,) ‘This completes the proof of (a) () Recalt the definition of OW, ve have the following (eect (8) equ to sep ft) ox cnpuet cee Tat ee rate eo es tana Tas ve can choose copact f,¢ F vth we) < HO) 1 and for each n , ve choose inductively @ K,,, with BaP Es CK U RU ee UR) Ey and 1 wG ante) Hemeis 0G, -K) 0, 2 By the proof of Riest's theoren,the measure y defined by ane 7)" () nO) sup fats eS 1 KEY, K compact { 2 BEV, V open | observe +p | cry" nce) «int fe 0D 1s a representing measure for A. . He thet af K=[x{x K', where K' is compact in R*, then by (*) for n and (#)" for minsteadof yu(8) is exactly a(K') (@ stands for Lebesgue measure on R*). Thus p is characterized by the identity wx] x Ja,bf) = bea, ER. This identity now gives a(RX[ 0) v= a5 follows : et V be an open set containing Rx }0|. Then for xR, (£50) € Vy s0 there exists an €,>0 with Va Lal gee ee This implies that there must be an n with uncountably aay ek. (HF ehiS isvnot the case, then €, 48 > 0 for at most countably many x, which contradicts the fact that R? is uncountable), bet pete EL fr eed rer w(K) is > B Kaku UK R, hence (¥) > supfB = 21,2, Thus we have proved that if V 4s an open set contain- fing Rx10{, then HCV) #405 by (OD, this implies s(Rxjoy) = =. Now if K ie a compact subset of Rx [01(K compact in the space 1), then Ke hence evigentiy qk) = 0. Therefore, for B= REx JO], ve find that ype th 10h WB) = mand sup |x(0): K compact, Kee] +0 thet i 4s not (inner) regular, QED. 20 CHERCTSE 35.~ 208 be a wettnordered uncourtebie ast vith iret atenent on for uy én By the net foes uczce| te denoted ty {ust] (and ainctarty fon other hinds of segnent)a bet wy = minfees + fas2,] te wncounsadie | It fottous that xm (a, wy] fe wellnordered, uncour- table, and for = ela, uj[ then [2,2] ta at most countable, For weK, tet By ala,e{ ond Sq = Ja, wy]. Coneider the topotogy on X generated by the By 'e and the Sq (the onder topotogy): Prove the folloving etatenente : ale) F be a compact RauedortP apace, (B) i} 9, | te not e-compact. (a) kot Bibs edlpant teed ef Ze then X ts ance! table iff wy tn a Linte point of &. (In other vords, every Sa contain’ a BOF hy 9 Cy

You might also like